Quantcast
  • Register
PhysicsOverflow is a next-generation academic platform for physicists and astronomers, including a community peer review system and a postgraduate-level discussion forum analogous to MathOverflow.

Welcome to PhysicsOverflow! PhysicsOverflow is an open platform for community peer review and graduate-level Physics discussion.

Please help promote PhysicsOverflow ads elsewhere if you like it.

News

PO is now at the Physics Department of Bielefeld University!

New printer friendly PO pages!

Migration to Bielefeld University was successful!

Please vote for this year's PhysicsOverflow ads!

Please do help out in categorising submissions. Submit a paper to PhysicsOverflow!

... see more

Tools for paper authors

Submit paper
Claim Paper Authorship

Tools for SE users

Search User
Reclaim SE Account
Request Account Merger
Nativise imported posts
Claim post (deleted users)
Import SE post

Users whose questions have been imported from Physics Stack Exchange, Theoretical Physics Stack Exchange, or any other Stack Exchange site are kindly requested to reclaim their account and not to register as a new user.

Public \(\beta\) tools

Report a bug with a feature
Request a new functionality
404 page design
Send feedback

Attributions

(propose a free ad)

Site Statistics

205 submissions , 163 unreviewed
5,047 questions , 2,200 unanswered
5,345 answers , 22,709 comments
1,470 users with positive rep
816 active unimported users
More ...

  Scalar field divergent mass correction interpretation question (hierarchy problem)

+ 3 like - 0 dislike
1002 views

Simple power counting tells you that a scalar field coupled to some fermions at one-loop picks up a correction to the mass of the order $\Lambda^2$.

Based on this people say things like "it's natural to expect that the mass of the scalar is roughly the cut-off scale", which in this case is some GUT/Planck scale.

My question is this: is this really the right interpretation? If I'm doing perturbation theory and it's telling me that I have a correction as big as the largest scale in my problem (cut-off scale), it means I cannot trust the answer. It does not meant the answer is $m_\phi^2 \propto \Lambda^2$. The renormalized mass could still be far below $\Lambda$, but the current approach cannot see that. The correct and finite answer might emerge only after adding up all diagrams. There's no reason to try to fine-tune anything such that already at one-loop the mass is small. One must simply concede that the one-loop answer is not correct.

What is the correct interpretation?

EDIT: corrected "far beyond $\Lambda$" with "far below $\Lambda$"

This post imported from StackExchange Physics at 2014-05-04 11:40 (UCT), posted by SE-user user129
asked Apr 26, 2014 in Theoretical Physics by user129 (5 points) [ no revision ]
retagged May 4, 2014
What makes you expect higher loop corrections to cancel a large one-loop contribution? Higher loops will have answers suppressed by higher powers of the coupling and so, will be parametrically suppressed. So for small values of the coupling, how can higher order corrections save you?

This post imported from StackExchange Physics at 2014-05-04 11:40 (UCT), posted by SE-user Siva

2 Answers

+ 0 like - 0 dislike

If I'm doing perturbation theory and it's telling me that I have a correction as big as the largest scale in my problem (cut-off scale), it means I cannot trust the answer. It does not meant the answer is $m_\phi^2 \propto \Lambda^2$. The renormalized mass could still be far beyond $\Lambda$, but the current approach cannot see that.

I disagree with this about a small point, but for now let's assume it is absolutely correct. Then you still have a scalar field that you would like to be massless but your calculation says its mass is of the order of $\Lambda$ or higher. This means that the hierarchy problem is still there and we are only arguing about a detail on how it is formulated.

Now the small point: It is actually very useful to know how the mass scales with the cutoff and there is a lot of information in knowing that $m_\phi^2 \propto \Lambda^2$ as opposed to for example $m_\phi^2 \propto \log\frac{\Lambda^2}{\mu^2}$ or anything else.

The way to think about it is this: Imagine another "fictitious" cutoff $\Lambda_f$ with $\Lambda_f\ll\Lambda$. Then your previous calculation will give $m_\phi^2 \propto \Lambda_f^2$, but now you are in a region where you can trust perturbation theory! Your calculation says that if you use to different fictitious cutoffs with $\Lambda_{f1}=2\Lambda_{f2}$ then the mass correction for the second theory will be 4 times bigger than the mass correction for the first theory.

Hope this helps!

This post imported from StackExchange Physics at 2014-05-04 11:40 (UCT), posted by SE-user Heterotic
answered Apr 26, 2014 by Heterotic (525 points) [ no revision ]
My original post was supposed to say "far below $\Lambda$", not "beyond". I meant to say that the true correction could be smaller, but we do not know.

This post imported from StackExchange Physics at 2014-05-04 11:40 (UCT), posted by SE-user user129
No, this cannot happen. If the true correction was much smaller than $\Lambda$, perturbation theory would work fine and give you the correct result. The fact that perturbation theory breaks down means you are guaranteed that the corrections are big. The only thing you can argue about is how big and if we can trust that they scale like $\Lambda^2$, but they will be big for sure. And this makes the hierarchy problem unavoidable.

This post imported from StackExchange Physics at 2014-05-04 11:40 (UCT), posted by SE-user Heterotic
+ 0 like - 0 dislike

It has to do with the scale to which you would like your physics to be valid. In general, we would like it to be correct up to a GUT scale. But in doing so, you would introduce a very large correction to mass of the scalar particle.

A simple dimensional analysis will show that if you go to higher order using the $\phi^4$ interaction then your diagrams will still be quadratically divergent, and there may be a cancellation depending upon the sign of the terms which will come next.

But there is no priori that there should be a cancellation of this very higher order and hence we have to say that it must be fine tuned to give sensible result.

This post imported from StackExchange Physics at 2014-05-04 11:40 (UCT), posted by SE-user user44895
answered Apr 26, 2014 by user44895 (35 points) [ no revision ]

Your answer

Please use answers only to (at least partly) answer questions. To comment, discuss, or ask for clarification, leave a comment instead.
To mask links under text, please type your text, highlight it, and click the "link" button. You can then enter your link URL.
Please consult the FAQ for as to how to format your post.
This is the answer box; if you want to write a comment instead, please use the 'add comment' button.
Live preview (may slow down editor)   Preview
Your name to display (optional):
Privacy: Your email address will only be used for sending these notifications.
Anti-spam verification:
If you are a human please identify the position of the character covered by the symbol $\varnothing$ in the following word:
p$\hbar$y$\varnothing$icsOverflow
Then drag the red bullet below over the corresponding character of our banner. When you drop it there, the bullet changes to green (on slow internet connections after a few seconds).
Please complete the anti-spam verification




user contributions licensed under cc by-sa 3.0 with attribution required

Your rights
...